The Student Room Group

AQA Physics PHYA4 - Thursday 11th June 2015 [Exam Discussion Thread]

Scroll to see replies

Reply 1680
Could someone explain this question please. I've come across it four times and have got it wrong repeatedly. Surely the gravitational potential at will be smaller than Y then at X as r is greater (V=(GM/r)) ?
Original post by JaySP
Could someone explain this question please. I've come across it four times and have got it wrong repeatedly. Surely the gravitational potential at will be smaller than Y then at X as r is greater (V=(GM/r)) ?



Potential is negative. So although with a bigger value of r, the magnitude of -GM/r is decreasing due to a larger value of r, it is becoming bigger because it is less negative. This is how I would take it. What's the answer to the question? I would've thought angular velocity is smaller at x as w=v/r and r is bigger at x.

I seem to remember getting this question wrong though, at least one of them is definitely the same from memory
Reply 1682
Original post by JJBinn
Potential is negative. So although with a bigger value of r, the magnitude of -GM/r is decreasing due to a larger value of r, it is becoming bigger because it is less negative. This is how I would take it. What's the answer to the question? I would've thought angular velocity is smaller at x as w=v/r and r is bigger at x.

I seem to remember getting this question wrong though, at least one of them is definitely the same from memory


Ah yes of course! Forgot potential was negative, thanks. The answer is B, and now that you've put the potential into perspective for me, it makes sense that the angular velocity is the same as they both have the same time period. Thank you.
Original post by JaySP
Ah yes of course! Forgot potential was negative, thanks. The answer is B, and now that you've put the potential into perspective for me, it makes sense that the angular velocity is the same as they both have the same time period. Thank you.


Yeah with the angular velocity I guess the key is that it says they are both points on the surface of the earth.

Also, with questions like that, if you are not sure I would go with the answer that appears most often in each column. For the first column, you know it will be ABC as there's 3 options the same and 1 different. Then column 2 50% of the answers are the same so it is likely to be one of the answers with this answer. And yes, it is B where they match up. Obviously this technique isn't official but it does work
Original post by JJBinn
Potential is negative. So although with a bigger value of r, the magnitude of -GM/r is decreasing due to a larger value of r, it is becoming bigger because it is less negative. This is how I would take it. What's the answer to the question? I would've thought angular velocity is smaller at x as w=v/r and r is bigger at x.

I seem to remember getting this question wrong though, at least one of them is definitely the same from memory


Yeah I've seen that question before and it's a bit confusing. In the question we do need to consider potential increasing as the distance increases from the body. But in fact potential is decreasing as the distance increases from the body according to how it's defined. The negative sign is an indication of the energy required similar to an exothermic reaction in chemistry. I still don't know why they consider potential being greater further from the body.


Posted from TSR Mobile
Reply 1685
Original post by Mehrdad jafari
Yeah I've seen that question before and it's a bit confusing. In the question we do need to consider potential increasing as the distance increases from the body. But in fact potential is decreasing as the distance increases from the body according to how it's defined. The negative sign is an indication of the energy required similar to an exothermic reaction in chemistry. I still don't know why they consider potential being greater further from the body.


Posted from TSR Mobile


This reminds me of our conversation the other day :wink:


Posted from TSR Mobile
Original post by CD223
This reminds me of our conversation the other day :wink:


Posted from TSR Mobile


Lol, yeah. To be honest i think potential at a point in space is not the work done per unit mass to be moved from infinity to that point. There is a lot that we don't know


Posted from TSR Mobile
Reply 1687
Original post by Mehrdad jafari
Lol, yeah. To be honest i think potential at a point in space is not the work done per unit mass to be moved from infinity to that point. There is a lot that we don't know

Posted from TSR Mobile


Haha. For the sake of the course though, that is the definition :wink:



Posted from TSR Mobile
Original post by CD223
This reminds me of our conversation the other day :wink:


Posted from TSR Mobile


There is also electric potential at subatomic level. So when an electron falls into the nucleus energy is released. I'm still not able to explain that


Posted from TSR Mobile
Reply 1689
Original post by Mehrdad jafari
There is also electric potential at subatomic level. So when an electron falls into the nucleus energy is released. I'm still not able to explain that


Posted from TSR Mobile


I suppose that kind of makes sense. Wouldn't be able to answer a question on it though.


Posted from TSR Mobile
Original post by CD223
Haha. For the sake of the course though, that is the definition :wink:



Posted from TSR Mobile


Lol, that's the truth, but a bitter truth :smile:


Posted from TSR Mobile
Reply 1691
Can someone explain what exactly we need to know? Other than the funny N and S shapes, when do they apply and what sort of situation?

ImageUploadedByStudent Room1432132402.118299.jpg

Is it limited to: current clockwise = S, current anti-clockwise = N?


Posted from TSR Mobile
Original post by CD223
Can someone explain what exactly we need to know? Other than the funny N and S shapes, when do they apply and what sort of situation?

ImageUploadedByStudent Room1432132402.118299.jpg

Is it limited to: current clockwise = S, current anti-clockwise = N?


Posted from TSR Mobile


I've never worked with S and N. is it to determine the direction of the magnetic field in a coil a result of a current flowing around it?


Posted from TSR Mobile
Reply 1693
Original post by Mehrdad jafari
I've never worked with S and N. is it to determine the direction of the magnetic field in a coil a result of a current flowing around it?


Posted from TSR Mobile


Yeah. N is anti-clockwise, S is clockwise :smile:


Posted from TSR Mobile
Reply 1694
Could someone help me understand why the resonant frequency occurs at a slightly lower frequency than the natural frequency, if the damping is not light?

I understand the sharp peak in amplitude being "damped", but can't understand why the frequency is affected if damping doesn't alter the time period?

ImageUploadedByStudent Room1432133781.147010.jpg


Posted from TSR Mobile
Original post by CD223
Yeah. N is anti-clockwise, S is clockwise :smile:


Posted from TSR Mobile


So when the current flowing clockwise it would produce a south magnetic field right?


Posted from TSR Mobile
Reply 1696
Original post by Mehrdad jafari
So when the current flowing clockwise it would produce a south magnetic field right?


Posted from TSR Mobile


That's correct. South Pole field is induced. Again, can't explain it beyond that as I'm just accepting what I'm being told :L


Posted from TSR Mobile
Original post by CD223
That's correct. South Pole field is induced. Again, can't explain it beyond that as I'm just accepting what I'm being told :L


Posted from TSR Mobile


Neither am I. But in the exam you don't need to know the magnetic pole induced of a coil carrying current. We will be told the direction of the current and the consequence of entering a magnet of known poles. With that and Lenz's law we can figure out the pole of the magnetic field of the coil.
Is that what you were asking?


Posted from TSR Mobile
Reply 1698
Original post by Mehrdad jafari
Neither am I. But in the exam you don't need to know the magnetic pole induced of a coil carrying current. We will be told the direction of the current and the consequence of entering a magnet of known poles. With that and Lenz's law we can figure out the pole of the magnetic field of the coil.
Is that what you were asking?


Posted from TSR Mobile


Yeah I was interested in how they could relate it to a question.

In Lenz's law, do we just say that a pole is induced that opposes the pole of the permanent magnet, causing two fields to repel and slowing a falling conductor down?


Posted from TSR Mobile
Original post by CD223
Yeah I was interested in how they could relate it to a question.

In Lenz's law, do we just say that a pole is induced that opposes the pole of the permanent magnet, causing two fields to repel and slowing a falling conductor down?


Posted from TSR Mobile


We might be asked to identify the poles but apart from that yeah, i think that's it.


Posted from TSR Mobile

Quick Reply

Latest

Trending

Trending